Inscription / Connexion Nouveau Sujet
Niveau terminale
Partager :

Module et argument, problème de résolution.

Posté par Pedrolito6 (invité) 06-05-05 à 11:57

Bonjour je suis lancé dans les révisions de mon baccalauréat et je n'arrive pas à trouver le module et l'argument de ce complexe:
z= \frac{\sqrt{3}-1}{2}(1-i)
Pour tout vous dire, je me suis lancé dans cette méthode en commencant par trouver l'argument:
cos(\theta)= \frac{\sqrt{3}-1}{2} = \frac{\sqrt{3}}{2} - \frac{1}{2} = cos(\frac{\pi}{6})-cos(\frac{\pi}{3})
sin(\theta)= \frac{-\sqrt{3}+1}{2} = \frac{-\sqrt{3}}{2} + \frac{1}{2} = sin(\frac{\pi}{6})-sin(\frac{\pi}{3})
Mais je ne sais pas comment avancer à partir de là, si quelqu'un pourrait m'aider je lui en serais reconnaissant. Pedrolito6

Posté par dolphie (invité)re : Module et argument, problème de résolution. 06-05-05 à 11:58

salut,

la première chose à faire est de déterminer le module!
Ecriture algébrique de z (de la forme z=x+iy).

puis |z|=\sqrt{x^2+y^2}

Posté par Samourai (invité)re : Module et argument, problème de résolution. 06-05-05 à 11:58

Essai de regarder les formules cos(a+b)=... et sin(a+b)=... Peut-être que ça donne quelque choses d'intéressant.

Posté par
isisstruiss
re : Module et argument, problème de résolution. 06-05-05 à 12:01

Bonjour Pedrolito6!

Pour calculer l'argument de z=a+bi, il suffit de calculer |z|=\sqrt{a^2+b^2}.

Isis

Posté par dolphie (invité)re : Module et argument, problème de résolution. 06-05-05 à 12:03

Toujours là au même moment!!!

Posté par
isisstruiss
re : Module et argument, problème de résolution. 06-05-05 à 12:03

Je me fais toujours dépasser par dolphie...

Isis

Posté par dolphie (invité)re : Module et argument, problème de résolution. 06-05-05 à 12:03

c'est marrant car en plus on dit tjs la même chose! (des fois il pourrait y avoir plusieurs résolutions...)

Posté par
isisstruiss
re : Module et argument, problème de résolution. 06-05-05 à 12:06

Ou plusieurs façons d'expliquer. Mais on a toujours les mêmes idées

Isis

Posté par Pedrolito6 (invité)Merci 06-05-05 à 12:07

Merci pour vos réponses rapides, je trouve:
|z| = \sqrt{(\frac{\sqrt{3}-1}{2})^2) + (\frac{1-\sqrt{3}}{2})^2)
Et en développant, j'en arrive à:
|z| = \sqrt{2-\sqrt{3}}
Si je n'ai pas fait d'erreur.
Je n'arrive pas ensuite à réutiliser ce résultat car il est sous une forme peu commune (deux racines). Merci encore de votre aide.

Posté par
isisstruiss
re : Module et argument, problème de résolution. 06-05-05 à 12:10

J'obtiens aussi le même résultat.

Isis

Posté par Pedrolito6 (invité)La phobie 06-05-05 à 12:15

On en arrive à ma grande peur, c'est un exercice type BAC de mon manuel et je serais desespéré de tomber sur ce genre de situation au BAC. Ce sont les questions classiques et il y en aura toujours qui seront tordu, on nous maltraite !!! Surtout que c'est la première question de l'exercice.

Posté par
isisstruiss
re : Module et argument, problème de résolution. 06-05-05 à 12:15

Tu peux aussi dire que |z|=\sqrt{2}\cdot\frac{\sqrt{3}-1}{2} si la double racine te gêne.

Isis

Posté par Pedrolito6 (invité)re : Module et argument, problème de résolution. 06-05-05 à 12:19

Oui mais sauf si je commet de grave erreur de calcul:
|z| = \sqrt{2}. \frac{\sqrt{3}-1}{2} = 3-\frac{\sqrt{2}}{2} Et donc ce n'est pas égale à |z|

Posté par
isisstruiss
re : Module et argument, problème de résolution. 06-05-05 à 12:26

|z|=\sqrt{2}\cdot\frac{\sqrt{3}-1}{2}=\frac{\sqrt{3}}{\sqrt{2}}-\frac{\sqrt{2}}{2}\neq3-\frac{\sqrt{2}}{2}

Tu peux voir ça plus facilement lorsque tu calcules le module:

\array{rll$z=\frac{\sqrt{3}-1}{2}(1-i)&\Rightarrow&|z|^2=\(\frac{\sqrt{3}-1}{2}\)^2+\(-\(\frac{\sqrt{3}-1}{2}\)\)^2=2\(\frac{\sqrt{3}-1}{2}\)^2\\ &\Rightarrow&|z|=\sqrt{2}\frac{\sqrt{3}-1}{2}

Isis

Posté par Pedrolito6 (invité)Exact 06-05-05 à 12:32

Pardon pour l'erreur de calcul et d'orthographe, j'ai tout à refaire. Bref, cela dit je n'avance toujours pas dans mon exercice, à partir de là il me faut maintenant avancer vers la solution pour que je puisse poursuivre mon exercice. Le problème à l'air conséquent, je continu de rechercher mais sans succès, j'attend toujours volontier une aide, merci encore.

Posté par
isisstruiss
re : Module et argument, problème de résolution. 06-05-05 à 12:37

Maintenant tu sais que

z=\frac{\sqrt{3}-1}{2}(1-i)=\frac{\sqrt{2}}{2}(\sqrt{3}-1}\(\frac{\sqrt{2}}{2}-\frac{\sqrt{2}}{2}i\)

Quel est l'angle tel que cos(\phi)=\frac{\sqrt{2}}{2}\qquad sin(\phi)=-\frac{\sqrt{2}}{2}?

Isis

Posté par Pedrolito6 (invité)Hum 06-05-05 à 12:45

Merci beaucoup, grâce à cette correction j'ai appris une nouvelle méthode pour déterminer un argument d'un complexe. J'ai déjà été piégé à un dst, ça ne m'arrivera plus. Je poursuis mon exercice, merci à toi isisstruiss. J'espere te redéranger le moins rapidement possible.

Posté par Pedrolito6 (invité)Réponse 06-05-05 à 12:47

L'angle c'est \frac{-\pi}{4}

***Edit Nightmare***

Posté par Pedrolito6 (invité)Merci 06-05-05 à 12:48

Posté par
isisstruiss
re : Module et argument, problème de résolution. 06-05-05 à 12:51

Oui, c'est bien ça. Mais avec ton erreur de balise je ne sais pas s'il y avait encore du texte dans ton message de 12:47. Il y a peut-être un modérateur qui va corriger ça à l'occasion.

Isis

Posté par
Nightmare
re : Module et argument, problème de résolution. 06-05-05 à 12:55

Il y a


Jord

Posté par Pedrolito6 (invité)C est bon 06-05-05 à 12:57

Il n'y avait qu'un smiley, celui-ci: . J'ai pris soin de rajouter car il était mérité.



Vous devez être membre accéder à ce service...

Pas encore inscrit ?

1 compte par personne, multi-compte interdit !

Ou identifiez-vous :


Rester sur la page

Inscription gratuite

Fiches en rapport

parmi 1675 fiches de maths

Désolé, votre version d'Internet Explorer est plus que périmée ! Merci de le mettre à jour ou de télécharger Firefox ou Google Chrome pour utiliser le site. Votre ordinateur vous remerciera !